TransWikia.com

Can I define a unitary representation of the Lorentz group on the Hilbert space of a theory which breaks Lorentz invariance?

Physics Asked on September 1, 2020

Let’s say I have some Lorentz-violating theory. For concreteness we could imagine a scalar field
begin{equation}
S=int {rm d}^4 x left(-frac{1}{2} (partial phi)^2 + V(x)^mu partial_mu phiright)
end{equation}

where $V(x)^mu$ is some space-time dependent vector field (ie a non-trivial external current sourcing $partial_mu phi$).

Classically, I can define a Lorentz transformation that acts in the normal way, $x^mu rightarrow Lambda^{mu}_{ nu}x^nu$, $phi(x) rightarrow phi(Lambda^{-1}x)$. Of course, this transformation won’t be a symmetry because the action will not be invariant.

What I want to understand is what the analog of this logic is in quantum field theory. Ie: I would expect that I could define a Lorentz transformation, but I should find there is some condition defining a symmetry that is violated by this transformation.

Now let’s say I quantize the theory, and work in the canonical formalism. Surely I can define the action of Lorentz transformations $Lambda$ on the Hilbert space. After all, given a state $|Psirangle$, I should be able to ask what the state looks like under a boost. Furthermore, I would expect the set of transformations to form a representation on Hilbert space, $R(Lambda)$, since the Lorentz transformations themselves form a group.

However, it’s not clear to me if the transformations can be taken to be unitary. I basically see two possibilities… (1) it is possible to construct a unitary representation of the Lorentz group on the Hilbert space of the theory, but it won’t have any physical relevance because the operators controlling the dynamics (4-momentum operator, angular momentum operator) won’t transform as tensors under these transformations, or (2) it is not possible to construct a unitary representation.

In favor of possibility (1): in non-relativistic quantum mechanics, I could always define angular momentum operators $L_i = epsilon_{ijk}x_j p_k$ which obey the algebra for $SU(2)$, even if the Hamiltonian is not Lorentz invariant, so it seems I could always construct a unitary representation of rotation operators by exponentiating these.

My questions are:

  • Is constructing $R(Lambda)$ possible in a Lorentz-violating theory? (If not, what stops you from constructing it?)
  • Can we choose a unitary representation for $R(Lambda)$ (possibility (1)), or not (possibility (2))?

2 Answers

If I understand the question correctly, then your question is more general than relativity. For example, you can ask the same question about rotations in a non-relativistic theory. In the spirit of trying to address the problem in the simplest situation possible, allow me to even downgrade to regular particle quantum mechanics, not a QFT. The Hamiltonian is of course, $$H = frac{textbf{p}^2}{2m} + V(textbf{x}).$$ Let's look at the rotation $R_{hat{n}}(theta)$, which is the rotation around the unit vector $hat{n}$ by an angle $theta$. The action of rotations on the Hilbert space is defined regardless of what is the Hamiltonian. For example we might simply define on the transformation operation on the Hilbert space as, $$U(R_{hat{n}}(theta))|textbf{x}rangle = |R_{hat{n}}(theta) textbf{x}rangle.$$ This is true whether the rotation is a symmetry of the Hamiltonian or not.

Whenever you meet a new operation, you need to figure out how it acts on the Hilbert space. Even though all symmetries of the Hamiltonian are unitary, not all unitary operations are symmetry of the Hamiltonian/Lagrangian. I hope this helps.

Correct answer by A. Jahin on September 1, 2020

If we want to describe the real world, QFT (quantum field theory) should respect the apparent symmetries of our universe. One symmetry is the translation invariance, another symmetry is the Lorentz invariance, which compound the Poincaré group ISO(1,3), the isometry group of Minkowski space. Formally $vert psi rangle to P vert psi rangle$ where $P$ is a Poincaré transformation. In addition unitarity is requested to guarantee that the matrix elements $M = langle psi_1 vert psi_2 rangle$ are Poincaré invariant. Under a Poincaré transformation we have $M = langle psi_1 vert P^dagger P vert psi_2 rangle$ asking for $P^dagger P = 1$.

The unitary representations of the Poincaré group are only a small subset of all of the representations of the Poincaré group. The unitary irreducible representations of the Poincaré group were classified by E. Wigner in 1939 and they are all infinite dimensional.

As for your question, I do not think it is a realistic description of nature to build a unitary representation out of a Lagrangian which is not Lorentz invariant. For instance in non-relativistic quantum mechanics you can have a particle spin state which is a linear combination of spin up and spin down and the norm is invariant under rotation, however in non-relativistic quantum mechanics you can not describe particles creation and annihilation.

Answered by Michele Grosso on September 1, 2020

Add your own answers!

Ask a Question

Get help from others!

© 2024 TransWikia.com. All rights reserved. Sites we Love: PCI Database, UKBizDB, Menu Kuliner, Sharing RPP